UTNianos

Versión completa: Ayuda 2do parcial AM2 [resuelto]
Actualmente estas viendo una versión simplificada de nuestro contenido. Ver la versión completa con el formato correcto.
Necesitaria verificar la resolucion de los puntos : P3 y P4

en el caso del P3 , la EC diferencial es Y"+Y= 0 en lugar de X

y en el P4 no tengo ni idea como se hace...
Te lo resuelvo .. y comparamos respuestas .. el T1 lo pienso je

T2) nos piden verificar el teorema de green entonces hay que plantear

\[\int_C fds=\iint_R Q_x-P'_y dA\]

para el primer termino de la igualdad parametrizo la curva que encierra la region como

\[g:R\to R^2/g(t)=(2\cos t,2\sin t)\quad t[0,2\pi]\]

la derivada

\[g'(t)=(-2\sin t,2\cos t)\]

entonces

\[\int_C fds=\int_{0}^{2\pi}(2\cos t+1,2\sin t-2)\cdot (-2\sin t,2\cos t)dt\]

haciendo las distributivas necesarias

\[\int_C fds=\int_{0}^{2\pi} -2\sin t-4\cos t dt=0\]

para el segundo miembro de la igualdad

\[\iint_R Q_x-P'_y dA=\iint_R 0 dA=0\]

por lo tanto se verifica el teorema de green

P1) nos piden el area de la esfera interior al paraboloide entonces la parametrizo como

\[g:R^2\to R^3/g(w,t)=(\sqrt{2}\cos w\sin t,\sqrt{2}\cos w\sin t,\sqrt{2}\sin w)\]

por definicion de area

\[A=\iint ||g'_u\times g'_v||dudv=\iint 2\cos w dwdt\]

no hay restricciones angulares en t. o sea que

\[t\in [0,2\pi]\]

la restriccion angular en omega esta dada por la ecuacion del paraboloide , evaluando esa ecuacion en nuestra parametrizacion obtenida y haciendo las cuentas se llega a

\[\sqrt{2}\sin w=2\cos^2 w=2(1-\sin^2w)\]

resolviendo la ecuacion se obtiene que

\[\sin w=\frac{\sqrt{2}}{2}\]

de donde se deduce que

\[\omega\in\left [ \frac{\pi}{4},\frac{\pi}{2} \right ]\]

finalmente

\[A=\int_{0}^{2\pi}\int_{\frac{\pi}{4}}^{\frac{\pi}{2}}2\cos w dwdt=(4-2\sqrt{2})\pi\approx 3.6806\]

verificalo con wolfram

P2) sale en cartesianas

\[V=\int_{0}^{1}\int_{x^2}^{\sqrt{x}}\int_{0}^{x+y}dzdydx=\frac{3}{10}\]

verificalo con wolfram

P3) el polinomio caracteristico de la ED dada es

\[r^2+1=0\to |r|=i\]

luego la curva es de la forma

\[y(x)=A\cos x +B\sin x\]

en los puntos por donde pasa la curva se cumple que

\[y(0)=0\quad y\left ( \frac{ \pi}{2} \right )=1\]

de donde finalmente

\[y(x)=\sin x\]

para la circulacion podes hacerlo de dos maneras

1) por definicion

\[\omega=\int_C fds\]

parametrizo la curva y la expreso de forma vectorial

\[g:R\to R^2/g(x)=(x, \sin x)\quad x\in \left [ 0,\frac{\pi}{2} \right ]\]

derivando y haciendo las cuentas respectivas queda

\[\omega=\int_{0}^{\frac{\pi}{2}} x-\sin x+2\sin x\cos x dx=\frac{\pi^2}{8}\]

verificalo con wolfram

2) cierro la curva C1 con otras dos auxiliares C2, C3 y aplico el teorema de green

\[\int_{C^+_1}=\iint_R-\int_{C^+_2}-\int_{C^+_3}\]

defino C2 como

\[C_2(x,0)\quad x\in \left [ 0,\frac{\pi}{2} \right ]\]

derivando y haciendo las cuentas correspondientes

\[\int_{C^+_2}=\int_{0}^{\frac{\pi}{2}}xdx=\frac{\pi^2}{8}\]

defino C3 como

\[C_3=\left ( \frac{\pi}{2},y \right )\quad y\in [0,1]\]

derivando y haciendo las cuentas

\[\int_{C^+_3}=\int_{0}^{1}2ydy=1\]

la integral doble se define como

\[\int_{0}^{\frac{\pi}{2}}\int_{0}^{\sin x} dydx=1\]

finalmente

\[\int_{C^+_1}=\iint_R-\int_{C^+_2}-\int_{C^+_3}=1-\frac{\pi^2}{8}-1=-\frac{\pi^2}{8}\]

pero nos piden que la curva vaya en sentido horario, entonces multiplicando por menos a toda la expresion

\[\int_{C^-_1}fds=\frac{\pi^2}{8}\]

P4) f esta definda por el gradiente de phi entonces

\[f(x,y)=(1,-2y)\]

te piden hallar las lineas de campo de dos maneras diferentes

1) por definicion de linea de campo

\[\frac{dy}{dx}=\frac{Q(x,y)}{P(x,y)}=\frac{-2y}{1}=-2y\]

de donde

\[\frac{dy}{y}=-2dx\]

integrando obtenes que la ecuacion general de las lineas de campo es

\[y=Me^{-2x}\]

2) las lineas de campo son perpendiculares a las lineas equipotenciales , las lineas equipotenciales permanecen constantes en cada punto de la funcion potencial entonces , tu funcion potencial es

\[\phi(x,y)=x-y^2=C\]

derivando implicitamente

\[1-2yy'=0\]

necesito las trayectorias ortogonales , entonces solo cambio y' por -1/y'

\[1-2y\left ( -\frac{1}{y'} \right )=0\to -2y=y'=\frac{dy}{dx}\]

integrando

\[y=Me^{-2x}\]

Pd: de que profesor es este parcial ??
Es el segun parcial que tomo cavallaron, lo aprobe pero me quedaron dudas con la nota que me puso la mina por que primero me dijo que promocione y cuando fui a firmar me mando a rendir un complemento de Ec diferenciales y lineas de campo,

hice basicamente lo mismo que vos, P1 y P2 me dieron lo mismo, el P3 me lo plantie y lo hice bien pero copie mal al pasar de una hoja a otra y llegue a otro resultado... y el P4 no lo sabia hacer, asi que voy a practicar ese tema ahora...

en cuanto a la teoria era enunciar los Teoremas , el T2 esta bien

y en el T1, lo plantie asi , tomando la superficie S como la union de S1uS2, entonces te queda la integral cerrada de circulacion de S1 + la integral de circulacion de S2 = 0 y como en la divergencia se toman las normales salientes , se te cambia el signo de las integrales y quedaria restando, cumpliendo :\[\iint_{S1} F. N1 DS1 = \iint_{S2} F. N2 DS2\]
URLs de referencia